The expected value for the binomial distribution with the provided probabilities is approximately 1.29888.
To find the expected value for the binomial distribution, we multiply each possible outcome by its corresponding probability and sum them up. In this case, we have the following outcomes and probabilities:
Successes: Probability:
0 243/3125
1 162/625
2 216/625
3 144/625
4 48/625
5 32/3125
To calculate the expected value, we multiply each outcome by its probability and sum them up:
Expected value = (0 * 243/3125) + (1 * 162/625) + (2 * 216/625) + (3 * 144/625) + (4 * 48/625) + (5 * 32/3125)
Simplifying this expression gives us:
Expected value = 0 + 0.26016 + 0.6912 + 0.27648 + 0.0384 + 0.03264
Expected value = 1.29888
For more such question on binomial. visit :
https://brainly.com/question/27990460
#SPJ8
I 3. A 6-m ladder is leaning against a vertical wall such that the angle between the ground and the ladder is 3. What is the exact height that the ladder reaches up the wall? ✓✓
The ladder reaches approximately 0.314 meters up the wall.
To find the exact height that the ladder reaches up the wall, we can use trigonometry.
Given:
The ladder has a length of 6 meters.
The angle between the ground and the ladder is 3 degrees.
Let's denote the height the ladder reaches up the wall as h.
In a right triangle formed by the ladder, the height h, and the base of the triangle (the distance from the wall to the ladder's base), we have the following:
sin(theta) = opposite/hypotenuse
sin(3) = h/6
To find h, we can rearrange the equation:
h = sin(3) * 6
Using a calculator, we can evaluate sin(3) to be approximately 0.05234.
Therefore, the height that the ladder reaches up the wall is:
h = 0.05234 * 6
h ≈ 0.314 meters
So, the ladder reaches approximately 0.314 meters up the wall.
Learn more about ladder from
https://brainly.com/question/29316136
#SPJ11
box plot
nobody gained exactly 30, 48 or 70 marks.
120 students gained less than 70 marks.
how many students gained more then 48 marks?
Considering the definition of quartiles,
Definition of quartilesQuartiles are measures that allow dividing values into equal parts and, based on that, locate the position of a given value. In other words, quartiles are the three values that divide an ordered data set into four equal parts. Therefore, the first, second, and third quartiles respectively represent 25%, 50%, and 75% of the statistical data set.
Then, the second quartile separates the data set into two halves and coincides with the median.
Number of students that gained more then 48 marksIn this case, the three quartiles are 30, 48, and 70, where Quartile 1 is of 30 marks, Quartile 2 is of 48 marks and Quartile 3 is of 70 marks.
So, the quartile 2 of 48 is the median of the data.
On the other hand, 120 students gained less than 70 marks or under the 1st and 2nd quartile.
According to the second quartile, 50% of the students (total) will score more than 48 marks, and 50% will score less than 48 marks.
Therefore, 120 students gained more than 48 marks.
Learn more about quartiles:
https://brainly.com/question/16983321
#SPJ1
Write a in the form a=a+T+aNN at the given value of t without finding T and N. r(t) = (2 e'√5)i + (2 e' cos t)j + (2 e' sin t) k, t=0 a(0) = (OT+N (Type exact answers, using radicals as needed.).
The answer is:a(0) = (2 e'√5)i + 6 e'j + 2 e'k.
The given vector function is:r(t) = (2 e'√5)i + (2 e' cos t)j + (2 e' sin t) k
Given value of t = 0a(0) = OT + N
To find a(0), substitute t = 0 in the given function r(t).
r(0) = (2 e'√5)i + (2 e' cos 0)j + (2 e' sin 0) kr(0)
= (2 e'√5)i + 2e'j
We need to write a in the form a = a + T + aN
where T and N are tangent and normal vectors of r(t) at t = 0 respectively.
Since a(0) = OT + N, we have to find T and N at t = 0.
Normal vector N can be given by:r'(0) = T(0) × N(0)As we have to find N(0), we need to find T(0) and r'(0).
Differentiating the function with respect to t, we get: r'(t) = -2 e' sin t j + 2 e' cos t k
Differentiating the above function with respect to t,
we get:r''(t) = -2 e' cos t k - 2 e' sin t j
Again differentiating the above function with respect to t,
we get:r'''(t) = 2 e' sin t j - 2 e' cos t k
We need to find r'(0) and r''(0) at t = 0.
At t = 0:r'(0) = -2 e' sin 0 j + 2 e' cos 0 k
= 2 e' kAlso,r''(0) = -2 e' cos 0 k - 2 e' sin 0 j= -2 e'j
Now, we can find T(0) and N(0)T(0) = r'(0) = 2 e' kN(0)
= T(0) × N(0) = (2 e' k) × (-2 e' j)
Now, we can find a(0).a(0) = OT(0) + N(0)a(0)
= r(0) = (2 e'√5)i + 2e'ja(0)
= a(0) + T(0) + aN(0)a(0)
= (2 e'√5)i + 2e'j + 2 e' k + 4 e' √5 i
To know more about vector visit:
https://brainly.com/question/30958460
#SPJ11
Help me i'm stuck w this 8
a) The exact volume of the mug is given as follows: 275π cm³.
b) Considering π = 3.14, the approximate volume of the mug is given as follows: 864 cm³.
How to obtain the volume of the cylinder?The volume of a cylinder of radius r and height h is given by the equation presented as follows:
V = πr²h.
The parameters for this problem are given as follows:
r = 5 cm, h = 11 cm.
Hence the volume is given as follows:
V = π x 5² x 11
V = 275π cm³.
V = 864 cm³.
More can be learned about the volume of a cylinder at brainly.com/question/9554871
#SPJ1
A study of 420.052 cell phone users found that 0.0309% of them developed cancer of the brain or nervous system. Prior to this study of cell phone use. the rate of such cancer was found to be 0.0317% for those not using cell phones. Complete parts (a) and (b). a. Use the sample data to construct a 95% confidence interval estimate of the percentago of cell phone users who develop cancer of the brain or nervous system, (Do not round unili the final answer. Thon found to throe decimal places as needed.)
A. The interval estimate is (0.02649%, 0.03531%).
B. We are 95% confident that the true percentage of cell phone users who develop cancer of the brain or nervous system falls within this range of values.
Part A: The interval estimate can be defined as a range of values that estimate the true population parameter.
The sample data represents a smaller section of the entire population. It is considered a sample and is used to represent the entire population. In order to calculate the interval estimate, the sample data is analyzed.
The interval estimate is then created to represent the true population parameter. The interval estimate provides a measure of confidence regarding the true population parameter.
The sample data from the study of 420.052 cell phone users found that 0.0309% of them developed cancer of the brain or nervous system.
This can be used to construct the interval estimate for the percentage of cell phone users who develop cancer of the brain or nervous system using a 95% confidence level.
To calculate the interval estimate, the following formula can be used:
p ± zα/2(√(p(1-p)/n))
where:
p = 0.000309
zα/2 = 1.96
n = 420,052
Plugging in the values, we get:
p ± zα/2(√(p(1-p)/n)) = 0.000309 ± 1.96 (√((0.000309*(10.000309))/420052)) = 0.000309 ± 0.0000441
So the 95% confidence interval estimate for the percentage of cell phone users who develop cancer of the brain or nervous system is:
0.000309 ± 0.0000441 = (0.0002649, 0.0003531)
Therefore, the interval estimate is (0.02649%, 0.03531%).
Part B: The interval estimate provides a range of values that estimate the true population parameter with a measure of confidence.
In this case, the interval estimate provides a range of values that estimate the percentage of cell phone users who develop cancer of the brain or nervous system with a 95% confidence level.
This means that we are 95% confident that the true percentage of cell phone users who develop cancer of the brain or nervous system falls within this range of values.
To know more about parameter refer here :
https://brainly.com/question/30896561
#SPJ11
Find the values of each of the angles marked in Fig. 13.15. 70⁰ 13.15 28
Suppose a restaurant has 4 possible entrées, chicken, beef, pork, and tofu, and the manager believes that each customer will independently order any one of them with probabilities 0.3,0.4, 0.2, and 0.1, respectively. When we ask for a distribution, please provide either a cumulative distribution, a mass function, a density function, or the name and parameter values for a standard distribution. a) Consider how many of the next ten customers will order the chicken entree. Find its distribution, expected value, and variance.
Let X be the number of customers who will order the chicken entree in the next 10 customers. Suppose a restaurant has 4 possible entrées, chicken, beef, pork, and tofu, and the manager believes that each customer will independently order any one of them with probabilities 0.3, 0.4, 0.2, and 0.1, respectively.
The probability that a customer will order the chicken entree is 0.3. The probability that a customer will not order the chicken entree is 0.7. Since each customer's order is independent, X follows a binomial distribution with parameters n = 10 and p = 0.3. Thus,X ~ B(10, 0.3).a) Distribution:
The probability distribution of X is given by:
P(X = 0) = (0.7)^10
= 0.0282P(X = 1)
= 10C1 (0.3) (0.7)^9
= 0.1211P(X = 2)
= 10C2 (0.3)^2 (0.7)^8
= 0.2335P(X = 3)
= 10C3 (0.3)^3 (0.7)^7
= 0.2668P(X = 4)
= 10C4 (0.3)^4 (0.7)^6
= 0.2001P(X = 5)
= 10C5 (0.3)^5 (0.7)^5
= 0.1029P(X = 6)
= 10C6 (0.3)^6 (0.7)^4
= 0.0367P(X = 7)
= 10C7 (0.3)^7 (0.7)^3
= 0.0090P(X = 8)
= 10C8 (0.3)^8 (0.7)^2
= 0.0015P(X = 9)
= 10C9 (0.3)^9 (0.7)^1
= 0.0002P(X = 10)
= (0.3)^10
= 0.0000
The mass function of X is given by:
f(x) = P(X = x)
where x = 0, 1, 2, ..., 10.b)
Expected Value:μ = E(X) = np = 10 x 0.3 = 3
Variance:V(X) = npq = 10 x 0.3 x 0.7 = 2.1
To know more about chicken visit:
https://brainly.com/question/12489069
#SPJ11
Find the integral sec-tan-dx 2 2 Do not forget the constant of integration. T (b) Find the area enclosed between the graph of y = cos(x), the x axis, the lines x = 4 π 3 Give the answer as an exact value. The results of any integration needed to solve this problem must be shown. and (c) Find the value of k such that - 3x² 0 -dx = ln217 x³ +1 Give the results of any integration needed to solve this problem. (d) Newton's Law of Cooling states that the rate of cooling of an object is proportional to the temperature difference between the object and its surroundings, if this difference is not too large. A bottle of juice at room temperature (22°C) is placed in a refrigerator where the temperature is 7°C. After half an hour the juice has cooled to 16°C. What is the temperature of the juice after another half hour? Give the results of any integration needed to solve this problem. (e) The number of organisms in a population at time t is denoted by x. Treating x as continuous dx xe+ dt 1+e where x is measured in variable, the differential equation satisfied by x and tis millions and t in hours. Initially x = 10. Find an expression for x in terms of t. Describe what happens to x over a long period of time. You must use calculus and give the results of any integration needed to solve this problem
The integral of sec-tan-x dx is -cosec(x) + C. The area enclosed between the graph of y = cos(x), the x-axis, and the lines x = 4π/3 is √3/2.
The integral sec-tan-x dx can be solved by using u-substitution in the following way.
Substitute u = sec x + tan x and du = (sec x tan x + sec² x) dx. We get,
∫sec x tan x dx = ∫du/u
= ln |u| + C
= ln |sec x + tan x| + C
The required integral is:
∫sec-tan-x dx = ∫(1/cos(x)) * (sin(x)/cos(x)) dx
= ∫sin(x)/cos²(x) dx
= -cosec(x) + C
Thus, the integral of sec-tan-x dx is -cosec(x) + C. The area enclosed between the graph of y = cos(x), the x-axis, and the lines x = 4π/3 is √3/2.
To know more about the integral, visit:
brainly.com/question/31433890
#SPJ11
An IQ test is designed so that the mean is 100 and the standard deviation is 12 for the population of normal adults Find the sample size necessary to estimate the mean IQ score of nurses such that it can be said with 99% confidence that the sample mean is within 3IQ points of the true mean. Assume that σ=12 and determine the required sample size using technology. Then determine if this is a reasonable sample size for a real world calculation. The required sample size is (Round up to the nearest integer.)
The required sample size to estimate the mean IQ score of nurses with 99% confidence and a margin of error of 3 IQ points is 107. This sample size ensures a high level of confidence in the accuracy of the estimate. However, whether this is a reasonable sample size for a real-world calculation depends on practical considerations and specific requirements of the study.
To estimate the mean IQ score of nurses with 99% confidence and a margin of error of 3 IQ points, we need to determine the required sample size. Given that the population standard deviation is σ = 12 and the desired confidence level is 99%, we can use the formula for sample size calculation.
The formula for sample size (n) in estimating the mean is:
n = ((Z * σ) / E)^2
Where:
- n is the required sample size
- Z is the Z-score corresponding to the desired confidence level (99%)
- σ is the population standard deviation
- E is the desired margin of error
First, we need to find the Z-score for a 99% confidence level. The Z-score can be obtained from a standard normal distribution table or using statistical software. For a 99% confidence level, the Z-score is approximately 2.576.
Plugging the values into the formula, we have:
n = ((2.576 * 12) / 3)^2
n ≈ (30.912 / 3)^2
n ≈ 10.304^2
n ≈ 106.12
Since we can't have a fractional sample size, we round up to the nearest integer. Therefore, the required sample size is 107.
This means we need a sample size of at least 107 nurses to estimate the mean IQ score with a 99% confidence level and a margin of error of 3 IQ points.
Determining if this is a reasonable sample size for a real-world calculation depends on various factors such as the available resources, time constraints, and practicality. In some cases, a sample size of 107 may be considered reasonable, while in other situations, a larger or smaller sample size may be preferred. Considerations such as the desired level of precision, variability within the population, and the importance of the estimation can also influence the determination of a reasonable sample size.
To know more about confidence intervals, refer here:
https://brainly.com/question/32546207#
#SPJ11
Find [tex] \tt \: \frac{dy}{dx} [/tex] when [tex] \tt {x}^{2} + {y}^{2} = log(x + y)[/tex]
Please help!
Answer:
[tex]\dfrac{\text{d}y}{\text{d}x}=\dfrac{1-2x^2-2xy}{2xy+2y^2-1}[/tex]
Step-by-step explanation:
Given equation:
[tex]x^2+y^2=\log(x+y)[/tex]
Assuming log(x + y) is the natural log:
[tex]x^2+y^2=\ln(x+y)[/tex]
To differentiate an equation that contains a mixture of x and y terms, use implicit differentiation.
Begin by placing d/dx in front of each term of the equation:
[tex]\dfrac{\text{d}}{\text{d}x}x^2+\dfrac{\text{d}}{\text{d}x}y^2=\dfrac{\text{d}}{\text{d}x}\ln(x+y)[/tex]
Differentiate the left side of the equation first.
[tex]\boxed{\begin{minipage}{4.5 cm}\underline{Differentiating $x^n$}\\\\If $y=x^n$, then $\dfrac{\text{d}y}{\text{d}x}=nx^{n-1}$\\\end{minipage}}[/tex]
Differentiate the terms in x only using the above rule:
[tex]2x+\dfrac{\text{d}}{\text{d}x}y^2=\dfrac{\text{d}}{\text{d}x}\ln(x+y)[/tex]
Use the chain rule to differentiate terms in y only.
In practice, this means differentiate with respect to y, and place dy/dx at the end:
[tex]2x+2y\dfrac{\text{d}y}{\text{d}x}=\dfrac{\text{d}}{\text{d}x}\ln(x+y)[/tex]
Now we have differentiated the left side of the equation, we can differentiate the right side of the equation.
[tex]\boxed{\begin{minipage}{6 cm}\underline{Differentiating $\ln(f(x))$}\\\\If $y=\ln(f(x))$, then $\dfrac{\text{d}y}{\text{d}x}=\dfrac{1}{f(x)}\cdot f'(x)$\\\end{minipage}}[/tex]
Apply the rule to differentiate ln(x + y):
[tex]2x+2y\dfrac{\text{d}y}{\text{d}x}=\dfrac{1}{x+y}\cdot \dfrac{\text{d}}{\text{d}x}(x+y)[/tex]
Differentiate (x + y):
[tex]2x+2y\dfrac{\text{d}y}{\text{d}x}=\dfrac{1}{x+y}\left(1+\dfrac{\text{d}y}{\text{d}x}\right)[/tex]
Simplify:
[tex]2x+2y\dfrac{\text{d}y}{\text{d}x}=\dfrac{1}{x+y}+\dfrac{1}{x+y}\dfrac{\text{d}y}{\text{d}x}\right)[/tex]
Rearrange the resulting equation to isolate dy/dx:
[tex]2y\dfrac{\text{d}y}{\text{d}x}-\dfrac{1}{x+y}\dfrac{\text{d}y}{\text{d}x}\right)=\dfrac{1}{x+y}-2x[/tex]
[tex]\left(2y-\dfrac{1}{x+y}\right)\dfrac{\text{d}y}{\text{d}x}=\dfrac{1}{x+y}-2x[/tex]
[tex]\left(\dfrac{2y(x+y)}{x+y}-\dfrac{1}{x+y}\right)\dfrac{\text{d}y}{\text{d}x}=\dfrac{1}{x+y}-\dfrac{2x(x+y)}{x+y}[/tex]
[tex]\left(\dfrac{2y(x+y)-1}{x+y}\right)\dfrac{\text{d}y}{\text{d}x}=\dfrac{1-2x(x+y)}{x+y}[/tex]
[tex]\dfrac{\text{d}y}{\text{d}x}=\dfrac{1-2x(x+y)}{x+y} \div \dfrac{2y(x+y)-1}{x+y}[/tex]
[tex]\dfrac{\text{d}y}{\text{d}x}=\dfrac{1-2x(x+y)}{x+y} \cdot \dfrac{x+y}{2y(x+y)-1}[/tex]
[tex]\dfrac{\text{d}y}{\text{d}x}=\dfrac{1-2x(x+y)}{2y(x+y)-1}[/tex]
To simplify further, expand the brackets:
[tex]\dfrac{\text{d}y}{\text{d}x}=\dfrac{1-2x^2-2xy}{2xy+2y^2-1}[/tex]
Answer:
[tex]\boxed{\bold{\tt\frac{dy}{dx}= \frac{2x^2+2xy-1}{1-2xy-2y^2}}}[/tex]
Step-by-step explanation:
x^2 + y^2 = log(x+y)
Differentiating both sides with respect to x.
[tex]\bold{\tt \frac{d}{dx} (x^2 + y^2) =\frac{d}{dx} log(x+y)}[/tex]
[tex]\bold{\tt{Apply\:the\:Sum/Difference\:Rule}:}[/tex]
[tex]\bold{\tt \frac{d}{dx}\left(x^2\right)+\frac{d}{dx}\left(y^2\right) =\frac{d}{dx} log(x+y)}[/tex]
Apply Power rule and chain rule
[tex]\bold{\tt2x+2y\frac{dy}{dx}=\frac{d}{dx} log(x+y)}[/tex]
[tex]\tt Apply\:the\:chain\:rule:[/tex]
[tex]\bold{\tt 2x+2y\frac{dy}{dx}= \frac{1}{x+y}\frac{d}{dx}\left(x+y\right)}[/tex]
[tex]\bold{\tt{Apply\:the\:Sum/Difference\:Rule}:}[/tex]
[tex]\bold{\tt 2x+2y\frac{dy}{dx}=\frac{1}{x+y}\frac{d}{dx}x+\frac{1}{x+y}\frac{d}{dx}*y}[/tex]
[tex]\bold{\tt 2x+2y\frac{dy}{dx}=\frac{1}{x+y}\frac{d}{dx}x+\frac{1}{x+y}\frac{d}{dy}*y*\frac{dy}{dx}}[/tex]
[tex]\bold{\tt 2x+2y\frac{dy}{dx}=\frac{1}{x+y}+\frac{1}{x+y}\frac{dy}{dx}}[/tex]
Solving for[tex]\tt \frac{dy}{dx}[/tex]
[tex]\bold{\tt 2x-\frac{1}{x+y}=\frac{1}{x+y}\frac{dy}{dx}-2y\frac{dy}{dx}}[/tex]
[tex]\bold{\tt\frac{1}{x+y}\frac{dy}{dx}-2y\frac{dy}{dx}= 2x-\frac{1}{x+y}}[/tex]
[tex]\bold{\tt\frac{dy}{dx}(\frac{1}{x+y}-2y)= 2x-\frac{1}{x+y}}[/tex]
[tex]\bold{\tt\frac{dy}{dx}(\frac{1-2y(x+y)}{x+y})= \frac{2x(x+y)-1}{x+y}}[/tex]
[tex]\bold{\tt\frac{dy}{dx}= \frac{\frac{2x(x+y)-1}{x+y}}{(\frac{1-2y(x+y)}{x+y})}}[/tex]
[tex]\bold{\tt\frac{dy}{dx}= \frac{2x(x+y)-1}{1-2y(x+y)}}[/tex]
[tex]\bold{\tt\frac{dy}{dx}= \frac{2x^2+2xy-1}{1-2xy-2y^2}}[/tex]
Therefore,Answer is:[tex]\boxed{\bold{\tt\frac{dy}{dx}= \frac{2x^2+2xy-1}{1-2xy-2y^2}}}[/tex]
Note: Formula
[tex]\boxed{\bold{\tt{Addition \: Rule:\frac{d}{dx}(x^n+y^n) =\frac{d}{dx}*x^n+\frac{d}{dx}*y^n}}}[/tex]
[tex]\boxed{\bold{\tt{Power \: Rule:\frac{d}{dx}x^n =n*x^{n-1}}}}[/tex]
[tex]\boxed{\bold{\tt{Chain \:\: Rule: \frac{d}{dx}y^n=\frac{d}{dy}y^n\frac{dy}{dx}=n*y^{n-1}\frac{dy}{dx}}}}[/tex]
[tex]\boxed{\bold{\tt{Product\:Rule:\frac{d}{dx}(u*v)=\frac{du}{dx}*v+u*\frac{dv}{dx}}}}[/tex]
15. Write down the form of a partial fraction decomposition for \( \frac{6 x^{3}-7 x^{2}+5}{(x-1)^{2}\left(x^{2}+3\right)} \). DO NOT SOLVE for \( A, B, C \), etc....
The given equation is:
[tex]\[ F(x) = \frac{6x^3 - 7x^2 + 5}{(x-1)^2(x^2+3)} \][/tex]
The form of the partial fraction decomposition for \( F(x) \) is:
[tex]\[ \frac{6x^3 - 7x^2 + 5}{(x-1)^2(x^2+3)} = \frac{A_{1}}{x - 1} + \frac{A_{2}}{(x-1)^{2}} + \frac{A_{3}x + B_{3}}{x^{2} + 3} \][/tex]
Note that the denominator of the function is already factored. The numerator has a degree less than the denominator as there is no term of degree 4 in the denominator, and the highest degree term in the numerator is of degree 3.
The first term of the partial fraction decomposition is due to the term \( [tex]\frac{1}{(x - a)^{n}} \[/tex]) in the denominator of the rational function, while the second term is due to[tex]\( \frac{1}{(x - a)^{n+1}} \)[/tex], and the remaining terms are due to the irreducible quadratic factors in the denominator, in this case, \( x^2 + 3 \).
If you further simplify the second term of the partial fraction decomposition above, you will get:
[tex]\[ \frac{A_{2}}{(x-1)^{2}} = \frac{B}{x - 1} + \frac{C}{(x - 1)^{2}} \][/tex]
The reason why we do this is that when it comes to solving for the constants, we can apply the method of undetermined coefficients and solve the resulting system of equations.
Hence, the form of the partial fraction decomposition for[tex]\( \frac{6x^3 - 7x^2 + 5}{(x-1)^2(x^2+3)} \)[/tex] is:
[tex]\[ \frac{6x^3 - 7x^2 + 5}{(x-1)^2(x^2+3)} = \frac{A_{1}}{x - 1} + \frac{A_{2}}{(x-1)^{2}} + \frac{A_{3}x + B_{3}}{x^{2} + 3} \][/tex]
To know more about partial fraction visit:
https://brainly.com/question/30763571
#SPJ11
Find the area, if it is finite, of the region under the graph of y=9x² e over [0,00). Select the correct choice below and, if necessary, fill in the answer box to complete your choice. (Type an exact answer.) A. The area of the region is B. The area is not finite.
The area of the region under the curve of the given function is not finite.The correct answer is option B.
Given function is y = 9x²e over [0, ∞).We need to find the area of the region under the curve of the given function. For this, we need to integrate the function over the interval [0, ∞).
The definite integral of a function f(x) over the interval [a, b] is given as: ∫aᵇ f(x)dxHere, the interval is [0, ∞). Therefore, we will write: ∫0^∞ 9x²e dx
Now, we will solve this integral. We will use integration by parts. Let u = 9x² and dv = e dx, then du/dx = 18x and v = eTherefore, we have: ∫0^∞ 9x²e dx
= [9x²e - ∫ 18xe dx]0∞
= [9x²e - 18xe + 18e]0∞
= [9x²e - 18xe]0∞
Since the limit does not converge, the area of the region is not finite. Hence, the correct answer is option B.
The integral of the function y = 9x²e over [0, ∞) can be evaluated using integration by parts. By taking u = 9x² and dv = e dx, we obtain du/dx = 18x and v = e.
On integrating by parts, we get [9x²e - 18xe]0∞. Since the limit does not converge, the area of the region is not finite.
:The area of the region under the curve of the given function is not finite.
To know more about integral visit:
brainly.com/question/31744185
#SPJ11
Is (9,6) a solution to the system of equation? y=-x-1 y=x-3
The point (9, 6) does not satisfy both equations simultaneously, it is not a solution to the system of equations y = -x - 1 and y = x - 3.
To determine if the point (9, 6) is a solution to the system of equations y = -x - 1 and y = x - 3, we can substitute the x and y values of the point into both equations and check if the equations hold true.
Substituting x = 9 and y = 6 into the first equation:
6 = -(9) - 1
6 = -9 - 1
6 = -10
The equation is not true, as 6 is not equal to -10.
Substituting x = 9 and y = 6 into the second equation:
6 = 9 - 3
6 = 6
The equation is true, as 6 is equal to 6.
For such more question on equations:
https://brainly.com/question/22688504
#SPJ8
Find The Surface Area Of Revolution About The X-Axis Of Y=2x+4 Over The Interval 1 ≤ X 1 ≤ 4.
Therefore, the surface area of revolution about the x-axis of y = 2x + 4 over the interval 1 ≤ x ≤ 4 is 44π√5 square units.
To find the surface area of revolution about the x-axis for the curve y = 2x + 4 over the interval 1 ≤ x ≤ 4, we can use the formula:
Surface Area = ∫[a,b] 2πy √(1 + (dy/dx)²) dx
First, let's find the derivative of y = 2x + 4:
dy/dx = 2
Next, let's evaluate the integral:
Surface Area = ∫[1,4] 2π(2x + 4) √(1 + 2²) dx
= ∫[1,4] 2π(2x + 4) √(5) dx
= 2π√5 ∫[1,4] (4x + 8) dx
= 2π√5 [(2x² + 8x)] [1,4]
= 2π√5 [(2(4)² + 8(4)) - (2(1)² + 8(1))]
= 2π√5 (32 - 10)
= 2π√5 (22)
= 44π√5
To know more about surface area,
https://brainly.com/question/31403430
#SPJ11
i need help bad please and thank you
(x; y-8)
The triangle moves down 8 spaces, so the y value decreases with 8. The triangle didn’t move left or right so the x value didn't change
Find The Maximum Profit And The Number Of Units That Must Be Produced And Sold In Order To Yield The Maximum Profit. Assume That Revenue, R(X), And Cost, C(X), Of Producing X Units Are In Dollars. R(X)=3x,C(X)=0.05x2+0.9x+9 What Is The Production Level For The Maximum Profit? Units
The production level for the maximum profit is 21 units.
To find the production level that yields the maximum profit, we need to determine the profit function and then find its maximum value. The profit function is given by:
Profit (P) = Revenue (R) - Cost (C)
Revenue (R) is given by the equation R(X) = 3X, where X represents the number of units produced and sold.
Cost (C) is given by the equation C(X) = 0.05X^2 + 0.9X + 9.
Substituting these equations into the profit function, we have:
P(X) = R(X) - C(X)
P(X) = 3X - (0.05X^2 + 0.9X + 9)
P(X) = 3X - 0.05X^2 - 0.9X - 9
To find the maximum profit, we need to find the critical points of the profit function. We can do this by taking the derivative of the profit function and setting it equal to zero:
P'(X) = 3 - 0.1X - 0.9 = 0
-0.1X + 2.1 = 0
-0.1X = -2.1
X = -2.1 / -0.1
X = 21
So, the critical point is X = 21.
To determine if this point is a maximum or minimum, we can take the second derivative of the profit function:
P''(X) = -0.1
Since the second derivative is negative, the critical point X = 21 corresponds to a maximum profit.
Therefore, the production level for the maximum profit is 21 units.
To know more about number click-
http://brainly.com/question/24644930
#SPJ11
Which rivalue represents the strongest correlation between the data and the equation? Select one: a. r=0.85 b. r=0.5 c 00.56 d. r= 0
The strongest correlation between the data and the equation is represented by the option (a) r = 0.85.
When we talk about correlation, we refer to the relationship between two variables. The correlation coefficient, denoted as "r," quantifies the strength and direction of the relationship. It ranges from -1 to +1, where -1 represents a perfect negative correlation, +1 represents a perfect positive correlation, and 0 represents no correlation.
In this case, a correlation coefficient of 0.85 (option a) indicates a strong positive correlation between the data and the equation. The closer the value of "r" is to 1 or -1, the stronger the correlation. Since 0.85 is closer to 1 than any of the other options, it represents the strongest correlation.
To know more about correlation coefficients, refer here:
https://brainly.com/question/29704223#
#SPJ11
A combination lock has three spinners with 7 letters and two spinners with 7 digits. How many possible codes are there using the 5 spinners. Show all work. (4 pt.) V9QC2 WORD3 X1SE4
Therefore, there are 16,807 possible codes using the 5 spinners.
To calculate the number of possible codes, we need to find the total number of combinations for each spinner and then multiply them together.
For the three spinners with 7 letters, each spinner can have 7 possible options (A, B, C, D, E, F, G). So, the total number of combinations for the letter spinners is 7 * 7 * 7 = 7^3 = 343.
Similarly, for the two spinners with 7 digits, each spinner can have 7 possible options (0, 1, 2, 3, 4, 5, 6). So, the total number of combinations for the digit spinners is 7 * 7 = 7^2 = 49.
Since the spinners are independent of each other, we can multiply the number of combinations for each type of spinner to find the total number of possible codes:
Total number of codes = Combinations for letter spinners * Combinations for digit spinners
= 343 * 49
= 16,807
Therefore, there are 16,807 possible codes using the 5 spinners.
Learn more about combinations here:
https://brainly.com/question/19692242
#SPJ11
Which of the following is equivalent to 34 · 3-2?
3 -8
3 8
3 2
3 -2
We can simplify the expression by multiplying the numerators and denominators:34 · 1/32 = (3 · 3 · 3 · 3)/(3 · 2 · 2 · 2)The common factors in the numerator and denominator can be cancelled: (3 · 3 · 3 · 3)/(3 · 2 · 2 · 2) = (3 · 3 · 3)/2 = 27/2The final result of the expression 34 · 3-2 is 27/2, which is equivalent to answer choice (C) 83.
The given expression is 34 · 3-2. We can simplify the expression by applying the exponent rule that states that a number with a negative exponent can be written as a reciprocal of the number with a positive exponent. Using this rule, we can rewrite 3-2 as 1/32. Therefore, we have:34 · 3-2 = 34 · 1/32
for more such question denominator
https://brainly.com/question/20712359
#SPJ8
three more than twice b
The equation we need to solve is:
3 + 2b = 13
And the solution is b = 5.
How to write this as an equation?Here we have the statement:
three more than twice b is equal to 13.
So we would want to write an equation and find the value of b.
"three more than..."
is written as
3 +
"...twice b is equal to 13"
3 + 2b = 13
That is the equation we want to solve.
subtract 3 in both sides:
2b = 13 - 3
2b = 10
b = 10/2
b = 5
Learn more about equations at:
https://brainly.com/question/22688504
#SPJ1
Complete question:
"Three more than twice b equal to 13?
disregarding the possibility of a february 29 birthday, suppose a randomly selected individual is equally likely to have been born on any one of the other 365 days. (a) if 11 people are randomly selected, what is the probability that all have different birthdays? (round your answer to three decimal places.)
The chance that 11 randomly selected individuals have distinct birthdays is about 68.8% (0.688), assuming no leap year and an equal probability for each day.
To calculate the probability that all 11 people have different birthdays, we can consider the scenario as follows:
The first person can have any of the 365 possible birthdays. The second person must have a different birthday than the first person, so there are 364 remaining possibilities. Similarly, the third person must have a different birthday than the first two, leaving 363 possibilities, and so on.
Therefore, the probability that all 11 people have different birthdays can be calculated as:
P(all different) = (365/365) * (364/365) * (363/365) * ... * (355/365)
Calculating this expression gives:
P(all different) ≈ 0.688
Rounded to three decimal places, the probability is approximately 0.688.Please note that this calculation assumes that the probability of being born on a particular day is the same for all individuals and ignores the possibility of a leap year.
To learn more about probability , click here:
brainly.com/question/31828911
#SPJ11
7. (2 pts) Determine whether the series converges or diverges. If it converges, determine whether the convergence is conditional or absolute. Show all steps and reasoning. n=1 (-1)" 1+3+5++ (2n-1)
Since the limit of [tex]b_{n}[/tex] is not 0, the series does not satisfy the first condition for convergence using the Alternating Series Test. Therefore, the series diverges.
the given series diverges and we do not need to determine whether the convergence is conditional or absolute.
The given series is:
∑([tex](-1)^n)(2n-1)[/tex]
To determine the convergence of this series, we will use the Alternating Series Test.
The Alternating Series Test states that if a series is of the form ∑([tex](-1)^n)b_{n }[/tex]or ∑([tex](-1)^{(n+1)})b_{n}[/tex], where [tex]b_{n}[/tex] > 0 for all n and [tex]b_{n}[/tex] is a decreasing sequence ([tex]b_{n}[/tex] > b_(n+1)), then the series converges if two conditions are met:
1. The limit of b_n as n approaches infinity is 0, i.e., lim (n→∞) [tex]b_{n}[/tex] = 0.
2. The sequence {[tex]b_{n}[/tex]} is decreasing, i.e., [tex]b_{n}[/tex] > b_(n+1) for all n.
Let's apply these conditions to the given series:
[tex]b_{n}[/tex] = (2n-1)
1. To check the limit of [tex]b_{n}[/tex] as n approaches infinity:
lim (n→∞) (2n-1)
= ∞ - 1
= ∞
To know more about infinity visit:
brainly.com/question/22443880
#SPJ11
Create a rational function that includes at least one asymptote, one zero, and one hole (all real numbers) for your classmates to analyze. Make sure to expand the numerator and denominator before you post your function. To analyze the function, find: (a) zero(s) (b) equations of the asymptotes (vertical, horizontal, and/or slant) (c) the coordinates of any hole(s) (d) y-intercept (if any) (e) End Behavior. Fill in the blanks: As x→[infinity],y→_________and x→−[infinity],y→____
rational function: y=(x+3)(x+1) /(x+1)(x-1)
(a) Zero(s): The rational function has one zero at x = -3.
(b) Equations of the asymptotes: The function has a vertical asymptote at x = -1 and a hole at x = 1.
(c) Coordinates of the hole(s): The function has a hole at (1, -4/2).
(d) Y-intercept: The y-intercept occurs when x = 0. Plugging x = 0 into the function, we get y = 3/1 = 3. Therefore, the y-intercept is (0, 3).
(e) End Behavior: As x approaches positive or negative infinity, y approaches 1.
Learn more about rational function
#SPJ11
Patients with chronic kidney failure may be treated by dialysis,
in which a machine removes toxic wastes from the blood, a function
normally performed by the kidneys. Kidney failure and dialysis can
cause other changes, such as retention of phosphorus, that must be corrected by changes in diet. A study of the nutrition of dialysis patients measured the level of phosphorus in the blood of several patients on six occasions. Here are the data for one patient (in milligrams of phosphorus per deciliter of blood)5.4 5.2 4.5 4.9 5.7 6.3The measurements are separated in time and can be considered an SRS of the patient’s blood phosphorus level. Assume that this level varies Normally with σ=0.9 mg/dl.
(a) Give a 95% confidence interval for the mean blood phosphorus level.
(b) The normal range of phosphorus in the blood is considered to be 2.6 to 4.8 mg/dl. Is there strong evidence that this patient has a mean phosphorus level that exceeds 4.8?
(a) The 95% confidence interval for the mean blood phosphorus level of the patient is between 4.182 and 6.452 mg/dl.
(b) There is no strong evidence that this patient has a mean phosphorus level that exceeds 4.8 mg/dl.
(a) To get a 95% confidence interval for the mean blood phosphorus level, we can use the formula:
CI = X ± t (s/√n)
Where,
X is the sample mean of the six measurements
s is the sample standard deviation of the six measurements
n is the sample size (which is 6 in this case)
t* is the t-score for the 95% confidence level with n-1 degrees of freedom
Plugging in the values we have, we get:
X = (5.4 + 5.2 + 4.5 + 4.9 + 5.7 + 6.3) / 6
= 5.3167
s = 0.726
n = 6
We can find the t-score using a t-distribution table or a calculator,
And for 5 degrees of freedom (n-1),
The t-score is approximately 2.571.
Plugging in all the values, we ge,
CI = 5.3167 ± 2.571 x (0.726/√6)
CI = (4.182, 6.452)
Therefore, we can say with 95% confidence that the true mean blood phosphorus level of the patient falls between 4.182 and 6.452 mg/dl.
(b) To test whether there is strong evidence that this patient has a mean phosphorus level that exceeds 4.8 mg/dl, we can set up the null and alternative hypotheses as follows:
Null hypothesis: The true mean blood phosphorus level is equal to or less than 4.8 mg/dl.
Alternative hypothesis: The true mean blood phosphorus level is greater than 4.8 mg/dl.
We can use a one-sample t-test to test this hypothesis.
The test statistic is calculated as:
t = (X - μ) / (s/√n)
Where:
X is the sample mean of the six measurements
μ is the hypothesized population mean (which is 4.8 in this case)
s is the sample standard deviation of the six measurements
n is the sample size (which is 6 in this case)
Plugging in the values we have, we get:
t = (5.3167 - 4.8) / (0.726/√6)
t = 1.933
Using a t-distribution table, we find the p-value associated with this test statistic to be approximately 0.054.
This means that if the true mean blood phosphorus level were actually equal to 4.8 mg/dl, we would expect to see a sample mean as extreme as 5.3167 about 5.4% of the time.
Since the p-value is greater than 0.05, we fail to reject the null hypothesis. Therefore, we do not have strong evidence that this patient has a mean phosphorus level that exceeds 4.8 mg/dl.
To learn more about statistics visit:
https://brainly.com/question/30765535
#SPJ4
Find (∂w/∂z) x
at (x,y,z,w)=(1,2,9,66) if w=x 2
+y 2
+z 2
+10xyz and z=x 3
+y 3
. ∗∗∗∗∗∗∗∗∗∗∗∗∗∗∗∗∗∗∗ w=x 2
+y 2
+z 2
+10xyz ve z=x 3
+y 3
olduğuna göre (x,y,z,w)=(1,2,9,66) daki (∂w/∂z) değerini bulunuz. A. 275
2
B. 4
275
c. 6
275
D. 3
275
E. 2
275
The value of the partial derivative [tex](dw/dz)_x[/tex] at (x,y,z,w) = (1,2,9,66) is 1,303.
To find [tex](dw/dz)_x[/tex] at (x,y,z,w) = (1,2,9,66),
First, we have to find the partial derivative of w with respect to z,
holding x constant.
Using the chain rule, we have:
⇒ dw/dz = (dw/dx) (dx/dz) + (dw/dy) (dy/dz) + (dw/dz)
To find (dw/dx), we take the partial derivative of w with respect to x, while holding y and z constant,
⇒ dw/dx = 2x + 10yz
And to find (dx/dz), we take the partial derivative of x with respect to z, while holding y constant,
⇒ dx/dz = 3x² + 3y²
Similarly, to find (dw/dy),
We take the partial derivative of w with respect to y, while holding x and z constant,
⇒ dw/dy = 2y + 10xz
And to find (dy/dz), we take the partial derivative of y with respect to z, while holding x constant:
⇒ dy/dz = 3x²+ 3y²
Finally, to find [tex](dw/dz)_x[/tex],
we substitute in the values from (x,y,z,w) = (1,2,9,66) and solve:
[tex](dw/dz)_x[/tex] = (dw/dx)(dx/dz) + (dw/dy)(dy/dz) + (dw/dz)
[tex](dw/dz)_x[/tex] = (21 + 10x2x9)(31² + 3x2² + (2x2 + 10x1x9)(3x1² + 3x2²) + 1
[tex](dw/dz)_x[/tex] = 1,303
Therefore, the value of [tex](dw/dz)_x[/tex] at (x,y,z,w) = (1,2,9,66) is 1,303.
To learn more about derivatives visit;
https://brainly.com/question/29144258
#SPJ4
Find the expected value of the winnings from a game that has the following payout probability distribution:
payout -2 0 2 4 6
probability 0. 67 0. 22 0. 07 0. 03 0. 01
expected value = ?
round to the nearest hundredth
The expected value of the winnings from the game is -1.02 (rounded to the nearest hundredth).
To find the expected value, we multiply each possible payout by its corresponding probability and sum up the results.
Expected value = (-2)(0.67) + (0)(0.22) + (2)(0.07) + (4)(0.03) + (6)(0.01)
Expected value = -1.34 + 0 + 0.14 + 0.12 + 0.06
Expected value = -1.02
Know more about expected value here:
https://brainly.com/question/28197299
#SPJ11
A manufacturer makes three models of a television set, model A, B, and C. A store sells 40% of model A sets, 40% of model B sets, and 20% of model C sets. Of model A sets, 3% have stereo sound; of model B sets, 7% have stereo sound; of model C sets, 9% have stereo sound. If a set is sold at random, find the probability that it has stereo sound.
The probability of stereo sound of a randomly selected set is 0.058 or 5.8%.
The given data is: Manufacturer makes three models of a television set: model A, B, and C.40% of Model A sets are sold.40% of Model B sets are sold. 20% of Model C sets are sold. 3% of Model A sets have stereo sound.7% of Model B sets have stereo sound. 9% of Model C sets have stereo sound.
The probability of the stereo sound of a randomly selected set is asked.
The probability of the stereo sound of a randomly selected set can be found by adding the probability of stereo sound of each model of the set sold multiplied by the probability of a set of that model being sold:
Probability of stereo sound of a randomly selected set = P(Model A) × P(Stereo Sound | Model A) + P(Model B) × P(Stereo Sound | Model B) + P(Model C) × P(Stereo Sound | Model C)
Let P(Model A) = probability of Model A being sold = 40/100 = 0.4
Let P(Stereo Sound | Model A) = probability of Stereo Sound given that Model A is sold = 3/100 = 0.03
P(Model B) = probability of Model B being sold = 40/100 = 0.4
Let P(Stereo Sound | Model B) = probability of Stereo Sound given that Model B is sold = 7/100 = 0.07
P(Model C) = probability of Model C being sold = 20/100 = 0.2
Let P(Stereo Sound | Model C) = probability of Stereo Sound given that Model C is sold = 9/100 = 0.09
Probability of stereo sound of a randomly selected set= P(Model A) × P(Stereo Sound | Model A) + P(Model B) × P(Stereo Sound | Model B) + P(Model C) × P(Stereo Sound | Model C)
= (0.4)(0.03) + (0.4)(0.07) + (0.2)(0.09)= 0.012 + 0.028 + 0.018
= 0.058
Therefore, the probability of stereo sound of a randomly selected set is 0.058 or 5.8%.
Learn more about probability visit:
brainly.com/question/31828911
#SPJ11
on a piece of paper graph y = -2X -4. then determine which answer matches the graph you drew
The graph of the equation y = -2x - 4 on a piece of paper shows a line with a negative slope.
The line intersects the y-axis at -4. The point where the line intersects the x-axis is (2,0). To graph the equation on a piece of paper, begin by marking the y-intercept at -4 on the y-axis.
Next, move to the right two units and down four units from the origin to mark the x-intercept. Finally, connect the two points with a straight line.
The equation y = -2x - 4 is in slope-intercept form. The slope of the line is -2, and the y-intercept is -4. To graph the equation on a piece of paper, begin by plotting the y-intercept at -4 on the y-axis.
Next, use the slope of -2 to find another point on the line. To do this, move down two units and to the right one unit from the y-intercept.
Continue to find more points on the line using the same slope until the line can be accurately drawn. Finally, connect all the points with a straight line.
for such more questions on graph
https://brainly.com/question/19040584
#SPJ8
Find the volume generated by rotating the region bounded by y=e=2, y = 0, I= - - 1, z = 0 about the line = 2. Express your answer in exact form. Volume=
The region bounded by y = e², y = 0, x = -1, z = 0 has to be rotated about the line x = 2. To find the volume of the solid obtained, we can use the cylindrical shell method. Volume = 2π(e⁴/2), which is approximately 38.472 units³.
We can start by sketching the region of integration and the axis of rotation. The region is a rectangle with height e² and width 2, so it looks like this:
We can see that the axis of rotation is at
x = 2, which means we need to shift the region to the left by 2 units.
The new region is shown below:
Now we can see that the integral that gives us the volume is:
V = ∫[2, 3] 2πx(e² - 0) dx
V = 2π ∫[2, 3] x(e²) dx
V = 2π [e²(x²/2) ] [2, 3]V
= 2π [e²/2] (3² - 2²)V
= 2π (e⁴/2)
Volume = 2π(e⁴/2), which is approximately 38.472 units³.
To know more about Region visit :
https://brainly.com/question/14573738
#SPJ11
A two-stage evaporator system is used to concentrate the 10% sugar solution to 50%. The feed stream is fed to the second stage at 21.1 C and the saturated water vapor at 110 C is fed to the first stage. A vacuum of 92.34 kPa is applied to the stage where the steam from the second stage is condensed. The heat transfer coefficients are U1=2271 and U2=1704 W/m2K. Heating surfaces of the same size are used in both stages. The boiling point elevation of the solution is negligible and the Cp= 3.98 kJ/kg.K for the solution.
(a) What are the heating surface areas?
(b) What is the individual and total economy of each tier?
(c) If the inlet and outlet temperatures of the cooling water used in the condenser are 15.5 and 60C, respectively, what is the flow rate? Feed rate is 4540 kg/hour.
(a) To calculate the heating surface areas for each stage of the two-stage evaporator system, we can use the equation:
Q = U*A*ΔT
where Q is the heat transfer rate, U is the heat transfer coefficient, A is the heating surface area, and ΔT is the temperature difference between the hot and cold streams.
For the second stage, we can use the feed stream temperature of 21.1°C as the hot stream temperature and the saturated water vapor temperature of 110°C as the cold stream temperature. The heat transfer rate can be calculated using the equation:
Q2 = (4540 kg/hour) * (0.1 kg/kg) * (3.98 kJ/kg.K) * (50 - 10)
Next, we can rearrange the equation to solve for A2:
A2 = Q2 / (U2 * ΔT2)
The temperature difference ΔT2 can be calculated as the difference between the feed stream temperature and the saturated water vapor temperature:
ΔT2 = 110 - 21.1
Similarly, for the first stage, we can use the saturated water vapor temperature of 110°C as the hot stream temperature and the cooling water outlet temperature of 60°C as the cold stream temperature. The heat transfer rate can be calculated using the equation:
Q1 = (4540 kg/hour) * (0.1 kg/kg) * (3.98 kJ/kg.K) * (110 - 50)
Again, rearrange the equation to solve for A1:
A1 = Q1 / (U1 * ΔT1)
The temperature difference ΔT1 can be calculated as the difference between the saturated water vapor temperature and the cooling water outlet temperature:
ΔT1 = 110 - 60
(b) The individual economy of each stage can be calculated using the equation:
Economy = Q / (m * h)
where Q is the heat transfer rate, m is the mass flow rate, and h is the enthalpy difference.
For the second stage, the heat transfer rate Q2 can be calculated using the equation from part (a). The mass flow rate m can be calculated using the feed rate of 4540 kg/hour and the mass fraction of the solution. The enthalpy difference h can be calculated using the specific heat capacity Cp and the temperature difference ΔT2.
Similarly, for the first stage, the heat transfer rate Q1 can be calculated using the equation from part (a). The mass flow rate m can be calculated using the feed rate of 4540 kg/hour and the mass fraction of the solution. The enthalpy difference h can be calculated using the specific heat capacity Cp and the temperature difference ΔT1.
The total economy of each tier can be calculated by summing the individual economies of the two stages.
(c) To calculate the flow rate of the cooling water used in the condenser, we can use the equation:
Q = m * Cp * ΔT
where Q is the heat transfer rate, m is the mass flow rate of the cooling water, Cp is the specific heat capacity, and ΔT is the temperature difference between the cooling water inlet and outlet.
Rearranging the equation, we can solve for the mass flow rate m:
m = Q / (Cp * ΔT)
The heat transfer rate Q can be calculated using the equation from part (a) for the first stage. The temperature difference ΔT can be calculated as the difference between the cooling water inlet temperature of 15.5°C and the cooling water outlet temperature of 60°C.
Know more about heating surface areas:
https://brainly.com/question/24209466
#SPJ11